D 2021 - the DI parts


This year's DI came in two parts of 90 minutes each. The DI had a well-balanced mixture of EPC and PCT questions, with common topics as well as some less familiar topics. 

Candidates that were well-prepared, with a sound legal knowledge and familiar with their legal reference books and other material should have been able to score 50-60% or more out of the 50 marks within the time available.

Some topics were expected, such as divisionals and (restoring) pendency; languages and translations – here, in PCT; priority, in particular some form of partial priority – here, in the form of a partial first application issue G 1/15, T 282/12; restoration of priority – here, recognition by the EPO as dO of restoration done by the USPTO as rO under the unintentional criterion; fees – here refunds of search and examination fees. Some topics were not so familiar, e.g., a PCT application filed with the EPO in English but with the text in the figures in Portuguese, and amendments of a granted patent in view of a national prior right in Germany (requiring discussions on Art.105a as well as on Art.123(2)&(3)).

The online access to the Guidelines GL/EPO and GL/PCT-EPO was convenient and may have been of additional help. E.g., the GL/EPO have an extensive part on refunds in GL/EPO A-X, 10, including links to RFees 9 and OJ 2019, A4 with the refundable amounts in 10.2.1 on refunds of search fees. As a PCT example, using GL/PCT-EPO was for me a very efficient way to find the R.26.3ter PCT provisions in GL/PCT-EPO A-V, 8 “Text matter in drawings” together with a paper copy of the PCT Rules, that gave all information needed to answer. I would have preferred hyperlinked pdf versions of the Guidelines rather than HTML pages, so that I could use them in the same way as during normal work, but the HTML version worked well too.

In earlier years, we have noted that the discussion was sometimes cut short already beforehand when we posted our full answers before candidates had the opportunity to post their comments and questions, and to discuss among them. Thereby, valuable feedback from candidates as to how they perceived certain questions, where they had difficulties, and where they had a different answer, and how that answer was different, did not always show up. For that reason, we did not post our answer immediately (the evening of 2 March), but we posted our answers to the DI parts the following morning (3 March, 10:45): the answers are at the end of this post. 

We look forward to your comments!
Comments are welcome in any official EPO language, not just English. So, comments in German and French are also very welcome!

Please do not post your comments anonymously - it is allowed, but it makes responding more difficult and rather clumsy ("Dear Mr/Mrs/Ms Anonymous of 02-03-2021 22:23"), whereas using your real name or a nickname is more personal, more interesting and makes a more attractive conversation. You do not need to log in or make an account - it is OK to just put your (nick) name at the end of your post.

Thanks!

NB: For our DII-blog, see here. For our first impressions blog, see here.

[Updated 04.03.2021 9:00:] We compiled a complete version of the D 2021 paper (both D1 parts, the DII part and the calendars): it is available here.  

----
[Update 3 March 2021, 10;45:]

Our answers to the DI questions


Note: EQE answers can be more concise than what we present below, as we sometimes have some redundant information allowing an easier understanding, and to also include information that will not attract marks but that you may have in our answer - you can then check whether your answer in in line with ours also for those parts. But: you need to provide a complete, explicit reasoning for full marks on a topic - any implicit step, skipped step or missing conclusion is a lost mark. Also note that our answers may also contain errors or omissions: any comments and discussion is much appreciated!

QUESTION 1 (9 MARKS)

Answer

  • This DI answer uses the legal texts as in force on 31.10.2020
  • The time limit to respond to the 94(3) expired:
  • 5/10/20 + 10d [R.126(2); 8/10 is within] + 4m [R.131(4)] -> 15/10/20 + 4m -> 15/2/21 (Mon)
  • So expired
  • as no response filed, appl is deemed to be withdrawn – Art.94(4)
  • and no longer pending today
  • But in January 2021, the period has not yet expired
  • So in January 2021, EP-A1 was still pending – R.36(1)
  • so EP-A2 was timely filed
  • But: divisional EP-A2 was filed in FR whereas EP-A1 was filed in EN
  • EN is official EPO language - Art.14(1) - and is the language of proceedings of EP-A1 – Art.14(3).
  • so EP-A2 is not filed in the correct language, as it should also have been filed in EN - R.36(2); G 4/08
  • so EP-A2 is not dealt with as an EP divisional – Art.90(2), J 13/14 (in Case Law Book II.F.3.3)
  • EP-A2 cannot be remedied
  • Cannot file a new normal EP application today, as that cannot claim priority anymore and Ep-A1 will already have been published, so that a new application would not be novel
  • So need to file a new divisional from EP-A1
  • but as that is not pending today, we first need to restore the pendency of EP-A1 – GL A-IV, 1.1.1
  • As 4m expired 15/2/21, still time to request FP as the time limit for doing so is 2m from notification of loss of rights which we did not yet get - so, we have at least 2 months
  • But it can be done immediately, no need to wait for communication if it hasn’t been sent yet.
  • So: Request  FP to restore pendency - Art.121, R.135 – by::
    • paying flat FP fee RFees 2(1).12
    • completing omitted act = respond to OA, e.g. by deleting invention Y from EP-A1
  • After pendency is restored:
  • file new divisional EP-A3 based in EP-A1
  • claiming and describing invention Y, and not claiming X
  • in English - R.36(2)
  • pay filing fee and search fee within 1m of filing - R 36(3)
  • EPO will then search invention Y

Q.1: Some non-standard aspects of divisionals such as effect of wrong language, some standard aspects such as restoring pendency.


QUESTION 2(8 MARKS)

Answer

  • 9/8/2019 was outside priority period 2/8/18 + 12m [Art.4C PC] -> 2/8/19 (Fri)
  • so not valid unless restored
  • 31m entry period runs from priority, even if not restored - R.26bis.2(c)(iii) and AG-IP 5.062 & Euro-PCT Guide 2.14.005[legal citation added 8 March 2021]:
  • 2/8/18 + 31m [R.159(1) EPC] -> 2/3/21 = today
  • So all R.159 EPC acts taken on last day of normal period for entry
  • Restoration by US/RO under R.26bis.3 used unintentional as criterion of restoration
  • but EPO uses all due care as criterion - Art.122 EPC;  GL E-IX, 2.3.5.3
  • so, EPO/DO does not recognize the restoration by US/RO - R.49ter.1(b) & R.49ter.1(e) PCT
  • So yes, need to take action before EPO as dO:
  • As 9/8/2019 is within 2m from the expiry of priority, can restore before EPO as dO – R.49ter.2(a)
  • Need to file a new request for restoration to EPO/DO - R.49ter.2(a)(i)
    • in writing
    • using and arguing all due care - GL E-IX, 2.3.5.3
    • within 1m from entry - R.49ter.2(b)(i), so 2/3/21 + 1m -> 2/4/21 (Good Fri) -> 6/4/21 (Tue)
    • stating reasons - R.49ter.2(b)(ii)
    • pay RE fee - R.49ter.2(b)(iii), RFees 2(1).13 in same time limit
  • EPO/RO will then restore

Q.2: Topic is perceived difficult by many candidates, but is well documented and studied/tested when studying. 

[Update 8 March 2021:]
Note: we added references to AG-IP 5.062 and 
Euro-PCT Guide 2.14.005 as R.26bis.2(c) itself is -as many PCT Rules- difficult to understand, while those two clarify the so-called "retention principle" well. However, usually, a reference to AG-IP or the Euro-PCT Guide does not attract marks; usually, a reference to the relevant PCT Rule, with the appropriate accuracy, is needed to score marks.

 

QUESTION 6 (8 MARKS)

Answer

1.
  • DE-G has an earlier effective date than EP-F and was published after the effective date of EP-F, and is a national right, so DE-G is a national prior right - Art. 139(2) EPC-, but not an EP prior right under Art.54(3)
  • As a national prior right, it is novelty destroying for DE only
  • Under R.138, different claims may be filed for DE in view of the national prior right DE-G:
    • must inform the EPO of its existence
    • file copy of DE-G, as evidence of a national prior right that justifies the limitation resulting in claims being restricted to DE and not all contracting states -GL-D-X, 10.1
    • can then do a limitation under Art.105a/b/c only for Germany - GL D-X, 10.1:
      • file request for limitation - Art.105a(1), R.92
      • with EPO
      • pay limitation fee – Art.105a(1), RFees 2(1).10a
      • give details about proprietor F and patent EP-F - R.92(2)(a) and (b)
      • file complete version of amended claim set, in the language of proceedings - R.92(2)(d)
      • indicate amendment and provide basis - R.137(4)
[Update 8 March 2021, in response to a comment from a reader:] The following comment may also be added and is expected to attract marks:
  • Although the text of the patent EP-F shall be uniform for all designated Contracting States - Art.118, last sentence-, it is allowed to file different claims for DE in view of the national DE prior right DE-G of the EPO is informed of its existence - R.138

2.
  • DE-G's "a food composition comprising 3% of meat, wherein the meat is pork" is a novelty destroying national prior right vs our claim "A food composition comprising 1 to 5% meat"
  • The claim can be amended for DE only in limitation - GL D-X, 10.1
  • Amending the claim to "A food composition comprising 1 to 5% beef meat" has basis in application EP-F as filed - Art.123(2), R.95(2)
  • It also limits the scope of protection from meat to beef meat - Art.123(3), R.95(2)
  • and is clear - Art,84, R.95(2)
  • and is, for DE, a limitation - R.95(2), GL D-X, 10.1
  • So that it appears that it can allowed and that we can get:
    • claim as granted (unamended) for EP except DE: "A food composition comprising 1 to 5% meat"
    • claim as amended for DE: "A food composition comprising 1 to 5% beef meat"
  • ButIn limitation, also Art.123(3) is to be complied with - R.95(2)
  • The proposed amended claim has a Art.123(3) problem: 
  • “A food composition comprising 1 to 5% beef meat” allows also other meat to be present, e.g., also some pork meat,
    • whereby the total meat content can be more than 5%,
    • so that it extends the scope beyond “A food composition comprising 1 to 5% meat” 
    • which violates 123(3).
    • GL H-IV, 3.4; T 2017/07 and T 287/11
  • So the proposed amendment for DE shall not be performed, but an alternative amendment has to be filed, only for DE:
  • Amend by introducing an undisclosed disclaimer G 1/03, G 2/16 in the claim - T 287/11
  • So submit:
    • claim as granted (unamended) for EP except DE: "A food composition comprising 1 to 5% meat"
    • claim amended for DE: "A food composition comprising 1 to 5% meat, with the proviso that the food composition is not a food composition comprising 3% of pork meat."
  • This satisfies Art.123(2), 123(3), 84, and it is a limitation of the claims – R.95(2), GL D-X, 10.1
  • [Update 4 March 2021, thx to a reader (see comments below):]
  • An alternative amendment for DE would be to amend it to a formulation ‘en cascade’:
    • claim for DE: “A food composition comprising 1 to 5% beef meat wherein the total amount of meat is 1 to 5%”
    • claim as granted (unamended) for EP except DE.
  • See Case Law book II.E.2.4.14 referring to, e.g., T 2017/07 and T 999/10:
  • This 'en cascade' claim does not have the 123(3) problem of the proposed claim as the total amount of meat is now limited to be 5%.
  • This satisfies Art.123(2), 123(3), and it is a limitation of the claims – R.95(2), GL D-X, 10.1

Q.6: Non-standard topic (limitation; and then in view of a national prior right). Note also as GL D-X, 10.1 does not unambiguously indicate whether you can have the different claims only as part of limitation of all claims (so limiting claims for all EPC states, and extra limitation for DE), or whether you can limit for one state only and keep the rest of the claims as they are. The latter is nowhere explicitly documented in the GL (as far as we are aware of). We assumed you can do it just for DE, based on GL D-X, 10.1. Also see H9-34(g) in our L-book (EQE 2021 edition).

The 123(3) problem was probably not spotted by many candidates:“A food composition comprising 1 to 5% beef meat” allows also other meat to be present, e.g., also some pork meat, whereby the total meat content can be more than 5%, so that it extends the scope beyond “A food composition comprising 1 to 5% meat”  which violates 123(3). There is case law on this (e.g. T 287/11 , on which there is Question H7-06 in our L-book; see also Case Law book II.E.2.4.14). That the proposed claim would not be allowable was hinted to in the question by “client proposes” – so that you need to double-check whether his proposal is OK and advice differently is needed- the possibility of another (also less restrictive) amendment…  An alternative amendment should be proposed: via an undisclosed disclaimer, for DE only.

See GL H-IV, 3.4: https://www.epo.org/law-practice/legal-texts/html/guidelines/e/h_iv_3_4.htm  :

A composition which is specified in a claim as comprising a component in an amount which is defined by a numerical range of values is subject to an implicit proviso excluding the presence of that component in an amount outside of that range. An amendment restricting the breadth of that component, for instance by narrowing down a generic class or a list of chemical compounds defining that component, has the consequence of limiting the scope of this implicit proviso. However, a composition which is defined as comprising the components indicated in the claim is open to the presence of any further components, unless otherwise specified. Therefore in a claim directed to such an openly defined composition, the restriction of the breadth of a component present therein may have the effect of broadening the scope of protection of that claim, with the consequence that in opposition/appeal proceedings such amended claim may extend the protection conferred by the granted patent (Art. 123(3)) (see T 2017/07 and T 287/11). Restricting the breadth of the component means that certain materials are no longer explicitly limited by the claim and therefore can be present in amounts which were excluded from the granted claim.

Note: Interesting that the first part had Q’s numbered 1, 2 and 6, rather than 1, 2 and 3 ;)


QUESTION 3 (7 MARKS)

Answer

EP-C1

  • Invitation to file claims just received
  • Such R.58 (jo. R.57(c)) invitation gives 2m
  • So period did not yet expire
  • as claims not yet filed, search did not yet start
  • So: withdraw EP-C1 before the expiry of the 2m period:
    • search fee paid for EP search shall be fully refunded - RFees 9(1), "withdrawn" - as the Office has not yet begun to draw up the search report; also GL A-X 10.2.1;
    • examination fee shall be refunded in full as withdrawn before substantive examination has begun  - RFees 11(a)-, as that can only start after the search and after a positive response to the R.70(2)
  • [Update 4.3.2021:] In stead of an active withdrawal, it is also possible to simply not respond to the R.58 invitation to get both fees refunded in full:
    • the application will then be refused - Art.90(5) - and the search will never start
    • so that the search fee paid for EP search shall be fully refunded - RFees 9(1), "refused",
    • examination fee shall be refunded in full  -RFees 11(a)- as refused fore substantive examination has begun
  • Applicant must, if not yet OK, update the refund instructions - so that the refunds are made either to a deposit account held with the EPO or by transfer to a bank account - OJ 2019, A26.

 EP-C2

  • Search already completed and search report issued, so no refund of search fee
  • As form 1001 was filed on filing, comprising the pre-printed request for examination, and the exam fee was already paid on filing, request for examination is deemed filed – Art.94(1)
  • As the R.70(2) was waived:
    • examining division became responsible when the search report was transmitted- R.10(4)- and
    • the search report was transmitted together with a Art.94(3) communication in view of the novelty objection - GL C-VI, 3
  • As a Art.94(3) communication is issued, substantive examination has already begun.
  • So withdraw the application before expiry of the time limit for replying to 94(3) communication (probably 4m) to get a refund of 50% of the examination fee - RFees 11(b)
  • Applicant must, if not yet OK, update the refund instructions - so that the refunds are made either to a deposit account held with the EPO or by transfer to a bank account - OJ 2019, A26.

Q.3: The two situations are original. RFees 9 and 11 are not very often tested. We also included the refund instructions in our answer, partially motivated by the explicit question “what do you have to do to obtain the refunds””and partially motivated by the recent updates to that in OJ 2019, A26.


QUESTION 4  (8 MARKS)

Answer

4a)

  • 8/20 EP-D0 published - bolt made of alu
  • 8/6/20 EP-D1 filed - bolt made of metal
  • 7/6/19 US-D filed - bolt made of metal
  • 2/19 EP-D0 - bolt made of alu
  • yes
  • irrespective of whether EP-D1's priority from US-D is valid or not:
    • EP-D0 has an earlier effective date than EP-D1: 2/19 vs 7/6/19 or 8/6/20
    • EP-D0 is published after the effective date of EP-D1: 8/20 vs 7/6/19 or 8/6/20
    • so that EP-D0 is 54(3) prior art vs EP-D1 in any case
  • the species bolt of alu of EP-D0 is novelty destroying for the genus bolt of metal in EP-D1

Note: we did not do the partial priority discussing in 4a) as it has no purpose here to do it in detail – that is done in 4c). G 1/15 also indicates that partial priority only needs to be assessed if there is a disclosure in the priority interval - G 1/15, reason 6.4.

4b)

  • yes
  • EP-D0 shows no other metals
  • so EP-D0 is only novelty destroyed by the bolt made of alu
  • alu is not disclosed in EP-D1
  • we can restore the novelty by adding an undisclosed disclaimer G 1/03 (or G 2/03), G 1/16 by amendment:
    • by disclaiming a bolt made of alu
    • such that novelty is restored over the 54(3) EP-D0
    • and no more is disclaimer than just that
  • such that is a disclaimer for legal erasons - G 1/03
  • which does not violate 12392) even though alu is not mentioned in EP-D1

4c)

  •  8/6/20 EP-D1 filed - bolt made of metal
  • 4/20 EP-D0 published - bolt made of alu
  • 7/6/19 US-D filed - bolt made of metal
  • 2/19 EP-D0 - bolt made of alu
  • 7/6/19 + 12m [Art.87(1)] -> 7/6/20 (Sun) [R.13491)] -> 8/6/20 (Mon)
  • so EP-D1 filed in time for valid priority
  • as both filed by D and identical contents so same subject-matter, prio seems valid for "a bolt made of metal"
  • But:
  • EP-D0 was filed by the same applicant in 2/19, so before US-D
  • EP-D0 is first application for bolt of alu
  • which is a species of bolt of metal
  • such that US-D is not the first application of the conceptual alu part of the metal-claim - G 1/15; T 282/12
    • whereas it is the first application for the conceptual remainder
  • Thus, EP-D1's priority from US-D is not valid for the conceptual part bolt of alu
    • whereas priority is valid for the conceptual remainder of the claim
  • Thus, the conceptual alu part of the claim has as effective date the filing date of EP-D1
    • whereas the conceptual remainder has as effective date the filing date of US-D
  • EP-D0 is thus a 54(2) prior art w.r.t. the conceptual alu part of the metal claim of EP-D1
  • as EP-D0 already show a bolt made of alu, the conceptual alu part of the claim is not new
  • and by that the whole claim to a bolt made of metal is not new
  • so yes, EP-D0 is novelty destroying in this case as well 
  • In this case, amendment by introducing a disclaimer cannot be used to restore the novelty as EP-D0 is not a 54(3) and alu is not described in EP-D1:
  • adding the disclaimer would violate 123(2)
  • no other amendments are possible as the application itself has no fallback
  • Also too late to add a valid priority claim to EP-D0

Q.4: Original. We expected a question on partial priority aspects in the DI in some form, either as G 1/15 itself (partial priority prio-subsequent) or as T 282/12 (partial first application very first-first-subsequent), and we have done T 282/12-like cases in various of our D courses; practice questions are also in our D-book. Also Mock 2 from the epi included a partial priority question... T 282/12 can be found in the Case Law Book.


QUESTION 5 (10 MARKS)

Answer

Situation A: PCT-E was filed in Portuguese.

(1) What is the receiving Office?

  • PCT-E was filed with EPO
  • but EPO as rO does not accept PT as a filing language, but only EN/FR/DE - R.12.1(a) PCT, R.157(2) EPC
  • so application is transmitted to IB - R.19.4(a)(ii) PCT
  • which accepts any language - AG-IP Annex C IB
  • and IB acts as rO - R.19.4(b)
[Update 8 March 2021, in response to a comment from a reader:] The following comments is most relevant for (3), but you could also validly add it here:
  • EPO is a competent rO for a PT national/resident - R.19.1(a)(i)/(ii) PCT, R.157(1) EPC

(2) What translation must be filed and by when?

  • PT is a publication language - R.48.3(a) -, so no translation needed for international publication
  • EPO was indicated as ISA and is a competent ISA for PT applicants - Art. 152 EPC
  • EPO as ISA does not accept PT, but needs EN/FR/DE translation - R.12.3(a), and EPO-WIPO Agreement 
  • applicant shall file that translation within 1m from receipt of PCT-E by the IB - R.12.3(a) & 12.3(c)(i)
    • 3/1/21 (when EPO received on behalf of RO/IB) + 1m [R.80.2] -> 3/2/21 (Wed), alreaady missed
  • or within 1m of the date of the invitation or 2m from the date of receipt by the rO - R.12.3(c)(ii), together with a late furnishing fee of 25% of the international fling fee - R.12.3(e)
  • so the later of:
    • 3/1/21 (when EPO received on behalf of RO/IB) + 2m -> 3/3/21 (Wed)
    • 29/1/21 + 1m -> 28/2/21 (Sun) [R.80.5] -> 1/3/21
  • So at the latest on 3/3/21

Note: legal basis for the EPO-WIPO Agreement is OJ 2017, A115 with several updates:
+ update Annex A(ii) [BE deleted as BE no longer rO] per 1/4/2018 - OJ 2018, A24
+ update Annex D, part I [new EPO fees] per 1/4/2018 – OJ 2018, A35
+ update Annex D, part I [new EPO fees & II(4) [Reductions] per 1/4/2020 – OJ 2020, A35
+ update Annex D, part I [fees] per 1/7/2020 – OJ 2020, A68


Situation B: PCT-E was filed in English, with the exception of the drawings, which contained text in Portuguese.

 

(3) What is the receiving Office?

  • PCT-E was filed with EPO
  • EPO as rO acepts EN as a filing language - R.12.1(a) PCT, R.157(2) EPC
  • EPO is competent rO for PT resident/national - R.19.1(i)/(ii) PCT, R.157(1) EPC
  • so EPO is the rO

 (4) What translation must be filed and by when?

  • As the description is in the correct language, English, no translation of PCT-E is needed under R.12.3(a)
  • The drawing annotations are not in English, which is a defect under Art 3(4)(i)
  • EPO/RO will invite the applicant to furnish a translation of the PT text matter of the drawings into the language in which the international application is
  • to be published, so in English - Rule 26.3ter(a) and GL/PCT-EPO A-V, 8:
    • within an (extendible) 2m time limit from the invitation - R.26.2 mut.mut. via R.26.3ter(a)
    • so by 29/1/21 + 2m -> 29/3/21 (Mon)
  • If missed, the office can extend this time limit of its own volition.

 Q.5: Original combi of R.19.4 and translations. We expect that not many candidates will have come to the correct and complete answer in 3&4 (PCT-E in English but drawings have text in Portuguese). Would have been easier if PCT and AG-IP would have been available online… 

(4) can be answered most efficiently via GL/PCT-EPO A-V, 8 in the Nov 2019 or March 2021 versionbut not in the Nov 2018 version (=syllabus on 31.10.2019). Without the GL/PCT-EPO (Nov 2019m or March 2021), candidates will have had difficulties to find R.26.3ter otherwise

Also, the Applicant's Guide provides the necessary information in AG-IP 6.018 (keyword Index -> Drawing(s) -> text matter in -- language of: 5.013-5.014, 6.014-6.020, 6.034).

Note that the Euro-PCT Guide (2020) in 2.23.001 indicates that also the drawings must be filed in EN/FR/DE if a PCT is filed with the EPO: “The international application, i.e. the request, description, claim(s), drawing(s) and abstract, must be filed with the EPO as receiving Office in English, French or German. An international application filed in another language will be forwarded to the IB to act as receiving Office instead of the EPO.” So candidates relying on this (usually very reliable) source from the EPO may have concluded that also in scenario B, the IB would be the rO.


Overall

In our opinion, D1 was somewhat more difficult than average. Good candidates will not have problems to score more than 50% of the marks, also when scoring low on one of the questions; weak candidates will score relatively lower than in an average D1. In our view, this is in line with what could be expected: it was announced that D1 would become more important (which was the reason to make the D1-D2 ratio different from the earlier 40:60).

In our opinion, D2 was somewhat less difficult than average. Good candidates will not have problems to score more than 50% of the marks, also when missing some aspects. Weaker candidates and poorly prepared candidates will have missed many aspects and many easy marks, due to incomplete reasoning, insufficient speed of answering or legal errors.

In our view, this was a well-balanced exam.


We look forward to our comments!


Please do not post your comments anonymously - it is allowed, but it makes responding more difficult and rather clumsy ("Dear Mr/Mrs/Ms Anonymous of 02-03-2021 22:23"), whereas using your real name or a nickname is more personal, more interesting and makes a more attractive conversation. You do not need to log in or make an account - it is OK to just put your (nick) name at the end of your post.


Roel, Diane, Stefanie

(c) DeltaPatents

Comments

  1. These are the topics as I understood them:
    Q1: divisionals, pendency of parent, compliance with 123(2) and filing of translation for divisional
    Q2: undisclosed disclaimers, G1/03
    Q3: 54(3) art, limitation
    Q4: refund of fees (Rfees A9(1) and Rfees A11)
    Q5: languages of PCT filing, IB as RO instead (A19.4((a) PCT), filing of translation of drawings when EPO is RO (R 26.3ter(a) PCT)... and more

    ReplyDelete
    Replies
    1. forgot Q2: late priority filing unintentional versus all due care (R 49ter.1(b) PCT)

      Delete
    2. Q3 was 139 (2) limitation. R 138 (2) different claims for different countrys

      Delete
    3. @2021 D: there were no translations of the divisional possible... the divisional was filed in the wrong language, so it is considered validly filed and gets no filing date. You need to file a new divisional in the correct language.

      Delete
  2. After all the technical glitches today, I don't feel like discussing my answers. Just want to have the answers from DP to compare and see :D
    ~ M

    ReplyDelete
  3. Was US-D and EP-D0 filed by the same applicant

    ReplyDelete
    Replies
    1. The question indeed does not explicitly indicate so, but why would you think it may not be the same? EP-D1 claims priority from it and it nowhere indicates that there was an issue with the applicants now being the same.

      If you consider it an uncertainty, then you need to address both situations in your answer. The "not-the-same-applicant" situation will be dealt with very quickly when you do so.

      Delete
    2. My question 4 and the Compendium's question 4 indicate that applicant D was the applicant for EP-D0 and EP-D1. The question doesn't indicate who filed US-D, or how D came to believe that s/he had a right to claim priority to US-D.

      If D wasn't applicant for US-D, and also did not acquire right to claim priority to US-D, then the priority isn't valid for EP-D1. Otherwise, the priority still isn't valid for EP-D1, because of EP-D0.

      What is the legal basis for priority being invalid if an applicant acquires the right to claim priority from an application (but not ownership of the application) that was filed by someone else after the applicant's initial filing?

      Delete
    3. The question indeed does not explicitly indicate so, but why would you think it may not be the same? EP-D1 claims priority from it and it nowhere indicates that there was an issue with the applicants now being the same.

      If you consider it an uncertainty, then you need to address both situations in your answer. The "not-the-same-applicant" situation will be dealt with very quickly when you do so.

      Delete
  4. My snapshot take:
    Q1. Divisional filed in wrong language. J13/14? Deemed not to have been received. Tough for Q1. FP parent and file new div in English.
    Q2. Restoration of priority before EPO as DO (R49ter.1 and .2) plus some usual re-establishment process
    Q6. Nasty. Limitation mixed with R138. Is it ok to only limit in one state? I presumed so. A colleague tells me there was a nasty A123(3) subtlety in composition comprising beef meat versus wherein the meat is beef meat
    Q3. Search & Exam fee Refunds. Felt like a lot of marks for not a lot. Unsure whether Art 94(3) accompanied search report for waived R70.
    Q4. (a), (b) Art 54(3) and fix by undisclosed disclaimer priority irrelevant for (a), (b). For part (c) priority matters. G1/15 applies, Al part not claimed from first application. Al part of claim not entitled to priority. Doc now 54(2) art against Al part of claim so cant disclaim.
    Q5. 1st bit RO=IB because filed in PT. File translation R12.3. 2nd bit. RO = EPO but translate drawing text. Some obscure corner of A14 / R26.3ter? PCT.

    ReplyDelete
    Replies
    1. Q2: Imo R49.ter2 procedure is after entry into EP regional phase, ie A150+A153+R159, and thus RE under A122/R136 should be cited. 2m term after expiry of priority period has already expired, so RE not possible.
      Q6: normally EP pat/appln is unitary - A118, but for the case of national prior rights proprietor can limit claims for just one CS - R138 (but evidence needs to be provided, there is a section in the guidelines on that)

      Q3: IMO the A 94(3) is issued by the examining division proper as it raises an objection and is accompanied by R71(1) inv, so only 50% reduction available (if appln withdrawn in time)

      Q4 a) was G1/15 + T282/12 „rationale of G1/15“ applies to first application assessment
      4b) G1/16 disclaimer (ok)
      4c) prior art bow A54(2), no effect on a) but disclaimer in b) not allowed anymore
      Agree with the rest.

      Delete
    2. Q6 the section in the guidelines: H-III 4.4

      Delete
    3. The deadline for restoration of priorty at EPO was 31 month deadline or 1 month after I.e. 32 months final deadline

      Delete
    4. Hi Oc, be careful: 1m after 31 is legally not the same as 32, and also practically not necessarily the same: if the 31m end in a weekend so extends to the next Monday, the 1m runs from the Monday!

      So you will -in my view- not get the marks for the time limit calculation if you do a single 32 m calculation: you need to do the 1m from the expiry of the 31m.

      Delete
    5. @Echo patents: Question 6:

      there was indeed a A123(3) subtlety in composition comprising beef meat versus wherein the meat is beef meat. See our answer and the cited GL-reference and case law therein.
      I wonder whether many candidates have seen it...

      Luckily, also when not having spotted that, there is still many marks to be obtained from that question.

      In my personal opinion, Question 6 was the most difficult of the exam (also because some other difficult topics were expected or at least candidates for topics this year, so you could/would have prepared fro those, such as partial priority)

      Delete
    6. Hi Roel van Woudenberg i do agree, but to be honest was cooking dinner at the time and thouught i should just pint it out in general.

      Delete
    7. @ Roel, I agree with your comments: I definitely missed the A 123(3) subtlety in Question 2 and found question 6 the most challenging.

      Delete
    8. I believe that even the person that drafted Q2 missed the subtle A123(3)... :)

      Delete
    9. @Roel Wow I did not spot that A123(3) issue in Q6! Very mean putting that in combination with an already very niche topic!

      Delete
    10. I do not think many candidates spotted that. But fortunately, there are still quite many marks to be obtained from the other aspects of Q.6.

      Delete
  5. You do not mention anything about the false start of DI. Even a brilliant prepared candidate, had extra stress and loss of focus in DI.

    Despite the lengthened exam time, scheduled breaks between the parts of Paper D were not extended, giving candidates limited time to recover from this rather chaotic start to the full day of examination. The break between DI and DII was not enough to relax a bit from the 4 hours in front of the computer between the start 9,30 CET and the end 13,20 CET of DI .

    I think the extra stress and loss of focus in the beginning of DI and the fact that scheduled breaks between the parts of paper D were not extended would affected the overall performance of all non-speaking German candidates which I deeply regret.
    (otherwise I spoted all main issues in DI/DII but I'm waiting to see your answers)Best regards

    ReplyDelete
    Replies
    1. I totally really agree with you, the start caused a lot of stress.
      But worst was the too short "lunchbreak" that affected me bad.

      Except that it was convenient to use computer instead of handwriting.
      I did a mistake in the afternoon stress and by spinal reflex tried to switch tabs with the normal shortcut-command. That caused that the lockdown browser locked and that I lost 15 minutes more to contact the Invigilators. They were very kind and understanding, but my unintentional mistake was a typical stress phenomena caused by the chaotic morning and the too short "lunchbreak".

      Delete
    2. Hi Patrik, I agree with you. I think I was more affected by the false start to D1.1 in D2 than in D1.1 or D1.2 because I didn't manage to have lunch in between the two parts... I had some snacks but was still starving halfway through D2 and could not focus very well anymore.

      Delete
    3. Fully agree... no words to express the frustration and loss of concentration after language blunder by EPO.

      Delete
    4. Rebecca MA, you are right - I felt the same - the false start of DI affected mostly DII

      Delete
    5. I also experiensed browser lock down at last 15 min. of DI part 1. Spent some desperate minutes trying to partly get browser to work again and dialog with invigilator. No success with browse, so I ended up in panic to write last arguments for Q2 in the chat with invigilator, hoping that I might be able to argue this in a later complaint. Since browser was out of play at end I was not able to hand in either, however invigilator ensured me the Secretariat would submit it automatically, I am not convinced. I ended part 1 pretty upset... Have tried to send an email to helpdesk@eqe and support@eqe, but not received any meaningful response yet. Hoping. Would be interesting to hear about how many had browser lock/hassle...

      Delete
  6. 26.3ter (a)(ii) PCT
    Where the abstract or any text matter of the drawings is filed in a language which is different from the language of the description and the claims, the receiving Office shall, unless

    the abstract or the text matter of the drawings is in the language in which the international application is to be published,

    invite the applicant to furnish a translation of the abstract or the text matter of the drawings into the language in which the international application is to be published. Rules 26.1, 26.2, 26.3, 26.3bis, 26.5 and 29.1 shall apply mutatis mutandis.

    => no invitation to file translation then

    ReplyDelete
    Replies
    1. Why not, it says "shall"!

      And further, the GL/PCT-EPO clarify the EPO practice in A-V, 8:

      "Where any text matter of the drawings is filed in a language which is different from the language of the description and the claims, the receiving Office will invite the applicant to furnish a translation of the text matter of the drawings into the language in which the international application is to be published (Rule 26.3ter)."

      (https://www.epo.org/law-practice/legal-texts/html/guidelinespct/e/a_v_8.htm)

      Delete
  7. Regarding the alu-metal question where EP-D0 is A54(2) prior art: It was indicated EP-D0 is in the same technical field, i also mentioned alu can not be disclaimed based on accidental anticipation criterion.

    ReplyDelete
  8. As always, thank you for all the work you do!

    A question about Q1: the question asked how to makee EPO search invention Y (or something along those lines).
    So in theory filing a further EP application disclosing and claiming invention Y would make EPO search invention Y (even if such invention wouldn't be novel vis-à-vis EP A1), thus correctly answering the question. Is this argument correct?

    ReplyDelete
    Replies
    1. It is a correct argument in the sense that the EPO will search Y in that case... but did you really consider that during the exam?
      I do not expect that there are any marks for such argument, as it ignores the context and the needs of the client. Also, you will already know what the EPO will find and you charged your client far over 1000 euros just for letting the EPO tell him/you that his already published EP-A1 already showed Y... and your client knows that too (and will not pay the bill).

      So, in the context of the whole question, the argument will not score any marks.

      Delete
    2. I considered both arguments...
      I said that it was advised to do as you suggested, but that in theory invention Y could be searched by EPO also by filing a non-divisional patent application claiming Y (although EP A1 would be novelty destroying for such claim).

      Now I fear that I'll get no marks as I provided two possible avenues (although 1 was preferred), and they'll consider only the worst one (as they do in some other papers)

      Delete
  9. Q1:
    Question was unclear at least in German, because “nunmehr” means “from that time point on / from now on” - so this is January 2021 then – the no reply was before and it was still time to file a new DIV without FP, as the 4 M period has not expired in January 2021. So instead of FP, the same points should be given if filing of the new DIV was done before expiring of the original 4 M period.

    Q3:
    For EP-C1 it should be also possible to just not respond to the R. 58 communication and let it be refused as it is still in formalities examination and therefore definitely before SR and start of examination

    ReplyDelete
    Replies
    1. I agree with your assessment of Q3, that is in my view be the "most correst" answer to the question

      Delete
    2. @Anonymous 3 March 2021 at 13:54

      Can you please post the complete Q.1 in the German version, so that we can understand your comment better?

      Delete
    3. @Roel: Below as I copied it out of the paper. Best regards Anja


      FRAGE 1 (9 PUNKTE)
      Die europäische Patentanmeldung EP-A1 wurde 2018 von Anmelder A in Englisch eingereicht. Sie beschreibt zwei verschiedene Erfindungen, X und Y, und beansprucht nur die Erfindung X. Der europäische Recherchenbericht wurde nur für die Erfindung X erstellt. Eine Mitteilung nach Artikel 94 (3) EPÜ mit einer gesetzten Frist von vier Monaten wurde am 5. Oktober 2020 vom EPA erlassen und von Anmelder A am 8. Oktober 2020 empfangen. Es wurde keine Erwiderung eingereicht. Anmelder A ist nunmehr an der Erfindung Y interessiert. Im Januar 2021 reichte er basierend auf EP-A1 eine europäische Teilanmeldung EP-A2 ein, welche die Erfindung Y beansprucht und beschreibt. EP-A2 wurde in Französisch eingereicht. Was sollte Anmelder A tun, damit
      die Erfindung Y vom EPA recherchiert wird?

      Delete
    4. Hi! I agree, it is ambiguous in German. I read this question at least 3 times and had never thought, that they meant "now" instead of "in January 2021". It would be great if this is considered when marking...

      Delete
    5. Can we complain about this ambiguity?

      Delete
    6. Sorry, I do not yet see how it changes the answer.

      Does it change the understanding of what "today" is (2 March 2021)?

      Does it change the interpretation of the subsequent facts:
      "Im Januar 2021 reichte er basierend auf EP-A1 eine europäische Teilanmeldung EP-A2 ein, welche die Erfindung Y beansprucht und beschreibt. EP-A2 wurde in Französisch eingereicht."?

      Does it change the interpretation of the subsequent question:
      "Was sollte Anmelder A tun, damit die Erfindung Y vom EPA recherchiert wird?"?

      By the way: if there is multiple valid interpretations, and the Committee knows about those and acknowledges those differences, they will be it into account in marking, and "an alternative answer" (based in the alternative interpretation) will also be marked according to its merits.

      A formal complaint had to be filed before the end of the exam day (see IPREE).
      You can submit informal comments by email to the EQE helpdesk, to the attention of the D committee and the Examination Board.
      Even more informally, you can clarity your point on this blog, and assume that they will read it.
      If you fail and believe it is due to this, you can file an appeal within one (1) months from your FAIL-decision, but you should note that the Disciplinary Board has only limited competence where it is about interpretation issues and marking (see the case law). So, better to prevent you will need a -likely unsuccessful- appeal and give your input on this blog and to the EQE secretariat as soon as possible.

      Delete
    7. For your reference, this is the wording in the English version of Q.1:

      QUESTION 1 (9 MARKS)

      European patent application EP-A1 was filed in 2018 by applicant A in English. It describes two different inventions, X and Y, and claims invention X only. The European search report was drawn up for invention X only.

      A communication pursuant to Article 94(3) EPC setting out a time limit of four months was issued by the EPO on 5 October 2020 and received by applicant A on 8 October 2020. No response has been filed.

      Applicant A is now interested in invention Y.

      In January 2021, he filed a European divisional application EP-A2 based on EP-A1, claiming and describing invention Y. EP-A2 was filed in French.

      What should applicant A do to have invention Y searched by the EPO?

      Delete
    8. Yes, it changes the understanding of "today" and further also changes the interpretation of the subsequent question: "Was sollte Anmelder A tun, damit die Erfindung Y vom EPA recherchiert wird?.

      As the question does not give "today" ("heute") and as the word “nunmehr” means “from that time point on / from now on / henceforward” this denotes that today is in January 2021. There is a difference in the meaning of "nun" (now) and "nunmehr" in German.

      As the wording of the question is:

      "Es wurde keine Erwiderung eingereicht. Anmelder A ist nunmehr an der Erfindung Y interessiert. Im Januar 2021 reichte er basierend auf EP-A1 eine europäische Teilanmeldung EP-A2 ein, welche die Erfindung Y beansprucht und beschreibt."

      it does not exclude, that still a reply can be filed, because "Es wurde keine Erwiderung eingereicht." is given before the subsequent sentence with "nunmehr" with the meaning = January 2021.

      So, in January 2021, the 4 M period has not expired yet and it was not necessary to request FP - therefore, due to this unclear wording, the part of the answer on FP was not necessary and not given.

      So question 1 would have needed the statement of "today" (heute) - like this is clearly given in question 2 ("Heute am 2. März 2021") or more precisely "No response has been filed AT ALL" (Es wurde ÜBERHAUPT keine Erwiderung eingereicht.) for being clear.

      Delete
    9. Thanks. But you seem to overlook that in the exam, "today" is the day of the exam (so, 2 March 2021). That does not need to be specified.

      How does that affect your interpretation?

      Delete
    10. NB: Unfortunately, the exam committees do not always specify in every question in D1, D2 and Pre-Exam what "today" is. Some tutors insisted on this for several years, and as a result of that, it is now usually done, but not always... However, it has also been indicated in several tutor meetings (and probably also in examiner's reports, but I did not check that) that "today" is the date of the exam unless explicitly otherwise indicated for a specific question.

      Delete
    11. But, there was no “today” (heute) in the wording of the question 1.

      So, due to the use of “nunmehr”, which in German refers to “from that time point on”, the time point is in January 2021, so from my understanding "today" is in January 2021.

      In contrast, in the English version it says “now” (which is “nun” in German), which can be seen directly as a byword for “today” and therefore March 2, 2021 can be assumed.

      Delete
    12. This too much on details of German grammar and German vocabulary for me to comment on...

      ..but a few general words as to interpretation differences and answers based thereon:
      in the exam, you are expected to take the date of the exam as the today-date in all questions, even if not stated.
      If a word "nunmehr" really changes that and puts the date back to January, and that reflects from your answer, I assume that they will take that into account int he marking.
      If the interpretation of ""nunmehr" putting you back in time will be considered an invalid or unreasonable interpretation, the question will be marked assuming that we are at "today".
      I think it is correct to say that a valid, but different interpretation that is not far-fetched will as a rule not make you miss marks - provided that it is clear from your answer what your interpretation was (which it will be in a well-reasoned answer).

      Delete
    13. That is the point: "nunmehr" sets the date to January 2021. In the Tutors‘ Report in epi 2019 it says: "When answering, “Today“ is the day of the exam." But it does not state that by default today is always the day of the exam. However, this is ambiguous, because of the wording used and because "today" was not mentioned in the Q1.

      Roel, thank you and the DP team very much for your assistance - you did an excellent job!

      Delete
    14. As said, my German is fair, but not good enough for this subtlety – I can only see that the way you present your arguments is quite convincing, "nunmehr" changing the meaning of today...

      Luckily, the effect of considering today to be in January is rather limit: your answer will then not have FP-to-restored-pendency argumentation, but will instead say something like "as today is jan 2021, the time limit to respond is still running so that EP-A1 is still pending, so we can file a new divisional EP=A3 today". The rest of the answer will be unaffected.

      If the Committee recognized that your interpretation of "nunmehr" defined "today" as being January, I can imagine that you may not even loose any mark.
      But if you nowhere indicated what today is in your view, and in view of today being in your interpretation different from what is normally is (=date of the exam), you may loose maybe 1-2 marks or so.

      NB: one argument against "today" being in January is that the Q says "No response has been filed" and not "No response has been filed yet": the first suggests that the time limit expired, the second that there is still time.

      Delete
  10. They will be lenient : covid situation, question in German, first time online for so many candidates....I pray that they will not remove too many points...2 key mistakes in D1 and 1 (half)-key mistake in D2 plus all the marks that I could not have scored...

    ReplyDelete
  11. One general question on 'In our opinion, D2 was somewhat less difficult than average'. How can you compare a 50 marks DII with the older 60 marks DII and conclude that it is less difficult?
    In my opinion, the difficulty can be measured by 1)- topics tested, 2)- book keeping of all the documents/dates.
    It may be that DII has less topics to test but the book keeping work was enormous.

    Have you considered this 50-60 marks difference in analyzing the difficulty level of DII?

    ReplyDelete
    Replies
    1. Agree, if we consider DII 2021 as a 50 mark paper I don't think that it was less difficult than average...

      Delete
    2. I also agree. Given the redistribution of marks, I expected D2 to be shorter than in the past. On the contrary, my timeline was at least as crowded if not more than in the past papers I practiced. I think the amount of information was the same if not more.

      Delete
    3. I think the length of the text was similar as before (roughly 4 pages), however there were less patents/applications to analyse (only three) and only one of them was pending, although everything was published years ago, so it was quite clear from the beginning that no additional applications could be filed and not much could be done such as adding priority claims etc

      Thus, I found the paper somewhat shorter than and less work than before...i was puzzling though that the situation of the client could not be improved as much as in previous papers

      Delete
    4. I agree with TT.
      The text had less applications, less events (so a less busy timeline) and less legal complexity than most of the papers of 2013-2019.
      Also less improvements, and the improvements you had to do were also less difficult than e.g. Art.61 or Art.55, or than restoration of priority under PCT in a DII.
      But having said that, surely the DII was challenging for most candidates (as an exam should be, isn't it?)

      Also, the paper helped you by providing abbreviations for all subject-matter, whereby your annotated timeline could be very concise.
      The available abbreviations also allowed you to answer using those - you could basically forget about the meaning of all of those, except that you had to check whether there was any genus-species relationship and expect that you needed to use and explicitly mention the effects when assessing inventive step.

      When comparing to earlier papers of 60 marks, the 50 marks and 3 hours (150 minutes + 30 minutes extra) for this paper thus seem quite OK to me (as would any number in a range of 45-52).

      Delete
    5. Thank you Roel for explaining more in detail what you meant!

      I see your point now and agree that improvements were relatively easy compared to previous years.

      Still think that there were many inventions to analyse (even if in fewer applications/patents than usually!)


      Delete
  12. QUESTION 3 (7 MARKS)

    Answer

    EP-C1: Imo, you actually don't need to take any steps, ie no active withdrawal is required, to get the search fee refunded. Reply to R. 58 is currently still outstanding, non-compliance will lead to refusal acc R. 57(c)+R. 58+A.90(5) - so the search fee will be refunded due to the refusal - A. 9(1) Rfees.
    You could also actively withdraw for the same result, but it's not required - and the question asked which steps would be required, if I remember correctly.



    Invitation to file claims just received
    Such R.58 invitation gives 2m
    So period did not yet expire
    as claims not yet filed, search did not yet start
    So: withdraw EP-C1 before the expiry of the 2m period:
    search fee paid for EP search shall be fully refunded - RFees 9(1) - as the Office has not yet begun to draw up the search report; also GL A-X 10.2.1;
    examination fee shall be refunded in full as withdrawn before substantive examination has begun, as that can only start after the search and after a positive response to the R.70(2)
    Applicant must, if not yet OK, update the refund instructions - so that the refunds are made either to a deposit account held with the EPO or by transfer to a bank account - OJ 2019, A26.

    ReplyDelete
    Replies
    1. I agree, two possible avenue to get total refund for EP C1: (i) active withdrawal, or (ii) do nothing at all (as you suggested)

      Delete
    2. Thx for your comments. You are correct. Not responding to the R.58 kleads to a refusal Art.90(5). For RFees 9(1) as well as for RFees 11(a), a refusal (before search resp. substantive examination started) is also good enough to get the full refund. We overlooked that in our initial answer, where we indicated that an active withdrawal before the end of the 2m period of R.58 was required.

      RFees 9(1) The search fee paid for a European or supplementary European search shall be fully refunded if the European patent application is withdrawn or REFUSED or deemed to be withdrawn at a time when the Office has not yet begun to draw up the search report.

      I updated our answer (see indication "Updated 4.3.2021)".

      Delete
    3. By the way, the benefit of an active withdrawal is that you get your money back faster if you request for the refund in the notice of withdrawal. The disadvantage of an active withdrawal is that you are bound by that (with basically no exceptions). If you discussed both options in your answers, you might have commented on some preference/advantage and may have advised which the applicant could do best (in Q.3, there seems to be no information that would drive you to a preference).

      Delete
  13. Thank you Roel and your colleagues at DeltaPatents for providing your answers! Now that I've read them, it's much easier to move on and focus on the next papers.

    ... and thank you also for alerting us about T 282/12 during your courses, you had a good gut feeling about it ;)

    I only have two comments to your answers:
    1) should the G decision in question 4 be G 1/03 instead of G 1/13?
    2) I based my answer to question 6 on GL D-X, 10.1 valid on 31.10.2019 and concluded that the EPO must be informed of the existence of DE-G but I neglected to specify that a copy of DE-G should be filed because the GL don't explicitly say it (of course, thinking about it now with a "fresh" mind, it makes sense but I didn't see it during the exam)

    Overall, I was relatively happy with the topics that came up in D1 even though I felt that more unusual topics came up than in the past, for example:
    - consequence of not filing a divisional in wrong language is not so straightforward - at least, it's not according to my reference book,
    - restoration of the priority right before the EPO as dO in question 2,
    - T 282/12 is a relatively niche T decision (it's not in the GL, is it?) and it would have been difficult to answer question 4 without it,
    - in question 5, I didn't find the reference to R26ter PCT as I was using the texts in force on 31.10.2019,
    - question 6 tested topics I hadn't encountered in any past paper.
    These feel like very many challenging decision points to face in a single paper (compared to past papers).


    I also know I missed some easy marks, e.g. in question 3 on the refund of fees, because I needed more time than in practise past papers to answer the more difficult questions (and because, let's be honest, I was a bit spaced out by then due to the prolonging of the exam time).

    Anyway, time to move on and focus on the next paper!

    ReplyDelete
    Replies
    1. Thx for your comments

      > 1) should the G decision in question 4 be G 1/03 instead of G 1/13?
      yes,typo, now corrected in the blog post.

      > 2) I neglected to specify that a copy of DE-G should be filed
      we added that as evidence will be needed; but that may also be filed after invitation if the indication of DE-G (e.g., its number) is there.

      Delete
    2. > - T 282/12 is a relatively niche T decision (it's not in the GL, is it?) and it would have been difficult to answer question 4 without it

      Indeed it is not. But also G 1/15 is hardly in it (just the headnote): you need more than just the G 1/15 headnote to understand partial priority.

      T 282/12 is in the Case law Book, so part of the syllabus...
      ...and candidates come to courses because courses guide them and help them understand the legal principle, the case law, and methodologies of/for exam papers ;)

      ...but also without T 282/12, you can come to the correct answer as -to use the wording of T 282/12, t.2.5 -:

      "2.5 In the Board's view, for reasons of consistency, the rationale of decision G 1/15 (concept of partial priority) must also apply in the context of deciding whether an application from which priority is claimed is the first application within the meaning of Article 87(1) EPC. Indeed, just as a priority application and a patent claiming priority therefrom may partially relate to the same invention, the priority application and an earlier application filed by the same applicant may also partially relate to the same invention. In that case, the priority application would be the first application in respect of only that part of the invention which is not the same as in the earlier application."

      Delete
    3. Interestingly, I did not find G 1/15 or T 282/12 but still answered the question correctly just from gut feeling and without citing any legal basis. Does anyone have an idea how this will be graded?

      Delete
    4. Difficult to say without seeing your complete answers - and without knowing how the committee will mark this question in detail.

      If you argued along the lines of argumentation of G 1/15 and T 282/1, and -e.g.- argued that Art.87(1) provides priority for what the priority document shows and does not require identity of that in the subsequent filing, I assume you will get quite some marks. But without an argumentation why claimed subject-matter benefits from different subject-matter in the priority, you will not get any marks I think.

      Delete
  14. @Q 5
    why for situation (a), rO is not IB or Portugese national office? they both accept Portugese. Translation is only required by the selection of ISA, i.e. EPO?

    ReplyDelete
    Replies
    1. Question 5 states that the application was filed *at the EPO*. So, the receiving office cannot be the Portuguese national office (it was not filed at the Portuguese national office). But since EPO does not accept Portuguese as a filing language, the EPO forwards the application to the IB which accepts all languages, so the rO is the IB. In cases of the wrong filing language, the office where the application was filed (in the wrong language) is sent to the IB.

      Delete
  15. Not sure if anyone had seen this already. There has been an announcement from EPO on the issue of DI - part I:

    http://documents.epo.org/projects/babylon/eponot.nsf/0/61E7BB0B4B091157C125868D0048FD6F/$File/message_from_the_eqe_examination_board_03.03.2021_en.pdf

    ReplyDelete
  16. Hi! why not remove c from S+o+c, and claim s+o with euro-PCT or divisional therefrom? Although s+o is not disclosed as such in euro-PCT, it seems that c can be removed as non essential and not inextricably linked to the other elements. This is evident from presence of s+o in the application of competitor. This also makes the joint applicant situation relevant, because a divisional must be filed by both applicants of mother application. What do you think? Eliminating an element from a claim was also a point in d paper of 2011 (bicycles), correct?

    ReplyDelete
    Replies
    1. See my answer to your same post in the DII blog.

      Delete
  17. Is there any Question paper itself so that non-attendance who is interested in [Pre-EQE guys here ;)] can see what are the questions? Thanks!

    ReplyDelete
    Replies
    1. Yes, it is available now: we compiled a complete version of the D 2021 paper (both D1 parts, the DII part and the calendars).

      It is available via the link in the blog post above (search for "Updated 04.03.2021)"

      Enjoy!

      Delete
  18. I actually didn't spot the partial priority point in Q4 (I just concluded that priority claim was invalid as US-D was not first application) - but it seems to me that this didnt effect the ultimate answer anyway i.e. in the part c situation EP-D0 is A54(2) and thus novelty destroying?

    ReplyDelete
    Replies
    1. Also thanks @Roel for the suggested answers, really useful!!

      Delete
    2. @Jonny: "this didnt effect the ultimate answer anyway"

      In the exam, marks are awarded for the reasoning: all explicit reasoning steps and the conclusion: you need to show that you come to the right conclusion in the right way. Typically, for an 8-mark question where the ultimate conclusion is a yes or a no, 7-7,5 marks go to the reasoning and only 0,5-1 to the "ultimate answer". So the number of marks missed when you did not address the key underlying principle is usually quite significant, even if the "ultimate answer" (not novel) seems correct.

      Delete
    3. Dear Roel,

      What happen if you do a right reasoning (citing one or more correct legal basis) but at the end answering the other way it should have done? It happened to me to another question, where I read the (PCT) rule in another sense, so at the end, I quoted to good one but my final answer is incorrect. I would guess that in that case, we obtain zero for the whole answer (meaning zero for the Yes or No part, and zero for the reasoning, since nothing is logical) It happened to me in D1.2 where probably, under the stress, I just misread the law...

      Also, the announcement of the Exam Committee about the fact that they are aware of the disruption in D1.1, do you think that they can add, for example, 5 marks to everyone ? I mean, I do not really understand this, because they were already aware during the exam, giving 30 minutes extra to everybody (although the German language problem last about 15 minutes, so for me, the exta 30 minutes were already a nice compensation...I actually did not complain after the exam and I also took the choice to use only 15-20 minutes of those extra 30 minutes for handing in my paper so to keep a bit of time for longer breaks. Personally, I would preferred having few free extra marks now, since also our breaks were disrupted, and our concentration were further disrupted for D1.2 and D2.... Indeed, in view of the proposed solution, it appears for me that D1.2 was harder than D1.1. Thanks in advance for your reply if any.
      Cheers

      StressPsycho (my nickname)

      Delete
    4. Dear StressPsych,

      If you do a right reasoning but at the end answering the other way it should have done, two situations an occur:
      - you made two conflicting and inconsistent reasonings (e.g., an argumentation that priority is correct and an argumentation that priority is not correct): then you score very low, as "alternative reasonings, one correct and one wrong, at the choice of the examiner do not get marks" (as a rule), as indicted in some tutor meetings;
      - if your reasoning was correct to a certain point and then you make an error whereby the rest is wrong (e.g., you do a correct and fully detailed priority analysis, and identify the prior art correctly, but you go wrong in the conclusion on novelty), you (as a rule) get the marks for the correct part.
      But it all depends on your answer as a whole and the details and quality (and understandability) of your answer, as well as to the type of errors that you make.

      best, Roel

      NB: Thx for using a Nickname and not posting it anonymously

      Delete
  19. We compiled a complete version of the D 2021 paper (both D1 parts, the DII part and the calendars).

    It is available via the link in the blog post.

    Enjoy!

    ReplyDelete
  20. Quite unfair to start with a question whose basis for answering can only be found in the decisions. I also can not really understand the logic behind: why is it possible to file a divisional in the original (non-official) language and afterwards filing a corresponding translation, and such procedure is not possible while filing the divisional in an offical language other than the language of proceedings?

    ReplyDelete
  21. W.r.t. Q.2, I got an email wherein it was proposed to also add the comment:

    "31m runs from priority date, even if not restored by rO – R.26bis.2(c)(iii)"

    I agree that that is a valid addition.

    ReplyDelete
  22. W.r.t. Q.6, I got an email wherein it was proposed to also add a reference to Art.118 EPC.

    I agree that that is a useful addition, even though I am not sure whether it will be needed for full marks, as there is already enough complexity in the question for the number of marks available.

    ReplyDelete
  23. W.r.t. Q.6, I got an email wherein it was indicated that, rather than a undisclosed disclaimer of which it may not be certain that that will accepted under German national law, the claim could be amended to read:

    “A food composition comprising 1 to 5% beef meat wherein the total amount of meat is 1 to 5%”

    which is sometimes referred to as formulation ‘en cascade’. See Case Law book II.E.2.4.14 referring to T 999/10.

    I agree that that is a valid alternative: it does not have an Art.123(3) problem.

    It will be interesting to see which possible amendments are all accepted by the Committee: the disclaimer, the formulation 'en cascade', other alternatives? Or maybe multiple independent claims for DE, so cover the scopes of both claim alternatives?

    In my view, the disclaimer version is the most preferred as it has the broadest scope (but the uncertainty of disclaimers for Art.139(2) and under DE law later on). But the formulation 'en cascade' provides the same scope of protection as what the client proposed, so that is also a good solution (smaller scope, but not those uncertainties).

    Thx a lot for drawing my/our attention to this amendment possibility!

    ReplyDelete
    Replies
    1. I have some doubts to this 'en cascade' proposal.

      The skilled person can only derive directly and unambigiously from EP-F the following subject-matter:
      a) "a food composition comprising 1 to 5% meat", from claim 1; and
      b) "a food composition comprising 1 to 5% beef meat”, from the description.

      It does not teach that all meat is beef meat.

      The amendment would therefore extend beyond the application as field and fails Article 123(2) EPC.

      Delete
    2. The 'en cascade' proposal does not indicate that all meat is beef meat.

      The wording "A food composition comprising 1 to 5% beef meat wherein the total amount of meat is 1 to 5%" includes having, e.g., 3% of beef meat and 1-2% of other meat, so a total of 4-5% of meat.
      But it excludes having, e.g., 3% of beef meat and 4% of pork meat, so a total of 7% of meat, so it does not have the 123(3) problem.

      Delete
    3. I have to admit that by the time I got to this question I was out of time due to the time lost with the language issue so had no time to analyse the claim wrt 123(3). Whilst I understand the logic presented for this I think from the wording of the question that the only support for any meat content in the application was that given in the question, I.e. that the only meat present was beef and that it was 1-5 % of the total composition. That would mean that the only amendment allowable to establish novelty over DE-G is one which defines that all the meat is beef. So something such as "A food composition comprising 1 to 5% meat wherein said meat is beef". Any claim which includes the possibility of pork (whether 3% or some other amount) would be added subject matter contrary to article 123(2) epc.

      Delete
  24. W.r.t. Q.5, I got an email wherein it was proposed to add to (1) that:

    "EPO is a competent rO for a PT national/resident - R.19.1(a)(i)/(ii) PCT, R.157(1) EPC."

    I agree that that is a valid and useful addition.
    We did not add it to (1) as the reason for the transmittal to the IB/RO is the language being wrong, and not the nationality/residence being wrong. Also, it does not effect the choice of the ISA being the EPO, as that would also have been the case if the RO was the PT office. But to be complete in (1), it would have been preferred to add it.

    The comment is however given in our answer to (3). I assume that that will be sufficient for getting the marks for the comment, also if it was expected for (1), as DI questions are usually marked as a whole (and it was not decisive for (1), as just explained - it may be different if it would have been).

    Thx for bringing up the point!

    ReplyDelete
  25. W.r.t. Q.5, I also got an email wherein it was proposed to add the reference to Art.14(1)(a)(v) PCT, as Rule 26 and its R.26.3ter are implementations of that article [to be precise: Art. 3(4)(i) gives the requirement, Art. 14 provides the legal framework as to what the receiving Office shall check, and Rule 26 implements Art.14(1)(a)(v)].

    I agree that adding such reference is useful, but based on the practice in the Examiner's Reports of the last several years, we assume that a reference to the Rule alone is sufficient to score the marks for legal basis (usually, only the "decisive" legal provision is cited, so usually a Rule if the Article is rather the umbrella but otherwise "empty, as here)"

    Thx for the email.

    NB: it is preferred if candidates post their comments directly on this blog, so that it can be commented on by other candidates as well without delay. But if you have difficulties to post comments (and the "Problems with commenting?" tips via the link in the right column of this page do not work), feel free to email me with your comments and I will post it on the blogs, as with this comments and the ones above)

    ReplyDelete
  26. In my opinion, taking into account what happened at the beginning of D1.1, the EPO should consider deleting one question from D1.1 per candidate (the one with lower marks).

    It may seem exaggerated, but the situation was unacceptable. Some canditates had 120 minutes, while other canditates had less than 75 minutes (leaving aside the extra stress and the increased possibilities of making mistakes).

    75 min were enough to fully respond at least two questions, but were clearly insufficient for responding three questions.

    The only way I see for "restoring the rights" of the candidates who were affected by the "disruption" is by eliminating one question.

    Is there any chance that the EPO would consider this?

    Thank you.

    ReplyDelete
    Replies
    1. See "Message from the Examination Board about paper D1.1 dated 3.3.2021":

      http://documents.epo.org/projects/babylon/eponot.nsf/0/61E7BB0B4B091157C125868D0048FD6F/$File/message_from_the_eqe_examination_board_03.03.2021_en.pdf

      "Munich, 3 March 2021

      Message from the Examination Board

      The Examination Board for the EQE is aware of a disruption affecting paper D1.1 and guarantees that no candidate will be disadvantaged as a result during the marking process."

      Delete
  27. Rule 27 IPREE: Examination Committee V shall be entrusted with the quality management of the papers. It shall advise the other Examination Committees in this respect.

    What did they do from April to July? From July to December? From December to 28 Feb? In the morning of 1 March? Since then?

    What will you write in the EPO Annual Reports of 2020 and 2021?

    ReplyDelete
  28. Hi Roel, thank you to Delta Patents for doing such a good work compiling the answers and commenting on the blog!
    Could you perhaps estimate how many marks Q4.c would be accorded? You can easily imagine why i am asking (unfortunately...🙂)

    Thanks again...

    ReplyDelete
    Replies
    1. I do not know...

      But if you insist on me to give a rough estimate, I would guess maybe 5 our of the 8 marks for Q.4. But could also be 4, 4.5, 5.5 or 6... (some of the reasoning of Q.4c is also needed for a and/or b, so some or it may marked there and if you then also miss it there, the effect of failing on Q.4c may be as high as 6 I can image).

      Delete
  29. In Q.1, I included that EP-A1 should not be amended (Y should not be deleted) before filing the new divisional EP-A3 as Y still needs to be in the parent application EP-A1 at the time the divisional EP-A3 is filed. That is missing on your proposed answer.

    Mike

    ReplyDelete
    Replies
    1. Hi Mike,

      Sorry, that is not correct: you CAN delete Y from EP-A1 before filing the divisional (as long as you do not do an explicit abandonment.

      Art.76(1) provides: "A European divisional application shall be filed directly with the European Patent Office in accordance with the Implementing Regulations. It may be filed only in respect of subject-matter which does not extend beyond the content of the earlier application AS FILED; in so far as this requirement is complied with, the divisional application shall be deemed to have been filed on the date of filing of the earlier application and shall enjoy any right of priority."

      NB: explicit abandonment is so uncommon and there is also no reason in Q.1 to do so, that there is no need to comment on that in your answer.

      Delete
  30. A candidate asked me in an email:

    I believe that the proposed claim is compliant with Art. 123(3), because its scope of protection is narrower that claim 1 of EP-F, so should also get full marks.

    ReplyDelete
    Replies
    1. This candidate seems to have overlooked that "A food composition comprising 1 to 5% beef meat" includes, for example, a food composition comprising 5% beef meat and 3% of pork meet, so a total of 8% of meet.

      So, it includes subject-matter that is not within the granted scope: "A food composition comprising 1 to 5% meat" does not cover food compositions with more than 5% of meet.

      Note also GL H-IV, 3.4: "A composition which is specified in a claim as comprising a component in an amount which is defined by a numerical range of values is subject to an implicit proviso excluding the presence of that component in an amount outside of that range."
      So, a composition comprising 1-5% excludes excludes that there is more meat than 5%.

      Delete
  31. The Examiners' Report is available!

    http://documents.epo.org/projects/babylon/eponot.nsf/0/7043D0BB750F0825C12586F8002C8336/$FILE/Compendium_ExRep_2021_D_EN.pdf

    The Possible Solution does not seem to have any “surprises” compared to our answers, except in Question (the third question of D1-1):

    a) The possible solution to Question 6-a) gives no explicit legal basis (except R.138) that it is allowed that a limitation can be done for Germany only, while leaving the claims unchanged for the other states.

    b) The possible solution to Question 6-b) (the third question of D1-1) only provides:
    “The proposed amendments are not allowable because the upper limit of 5% now applies to beef meat only. Therefore for example a food composition with 3% pork meat + 3% beef would fall within the scope of the amended claim but not in the claim as granted, Guidelines H-IV, 3.4.
    The request will thus be rejected, Art.105b(2) EPC as it does not meet the
    requirements of Art. 123(3) EPC, Rule 95(2) EPC”
    Surprisingly, it was not needed to indicate a correct form of the amendment. This may be an indication that only very few candidates came up with the correct amendment (the Report does indicate that “Very few candidates realized that the amendment proposed did not meet the requirements of Art. 123(3) EPC”).

    The Examiner's Report does not contain any comments w.r.t. the D1-1 issue (with only the German version being available at its start), nor any other-effects of the online format. It also does not indicate whether those have somehow been reflected in the marking.

    Please give your comments to the blogs posts for the D1-parts - please give the comments the D2-part to our D2 blog post (http://eqe-d.blogspot.com/2021/03/d-2021-dii-part.html)

    ReplyDelete
    Replies
    1. Lack of 31 October 2018 legal basis = unfair treatment19 June 2021 at 16:46

      It's extremely disappointing that the EPO tells candidates that they can use either the 31 October 2018 legal basis or the 31 October 2019 legal basis, but then only see fit to provide 31 October 2019 legal basis for the Examiner's report. As a candidate who was supposed to set EQE 2020, the EPO’s announcement that I could continue to use the 31 October 2018 legal basis allowed me to conserve the resources I had prepared for 2020, and came as a relief. However, now I feel both inconvenienced and penalized by the absence of 31 October 2018 legal basis from the Examiner’s report.
      First, the absence of 31 October 2018 legal basis from the Examiner’s report makes it difficult for candidates who used this legal basis in their answers to estimate their results. Incidentally candidates wouldn't need to estimate their results if the EPO had decided to issue the results alongside or before the Examiner's report, but that’s another matter….
      Second, the absence of 31 October 2018 legal basis from the Examiner’s report significantly increases the amount of time and effort candidates such as myself would have to spend if we needed to file appeals. Specifically, before such a candidate can determine whether s/he lost a point due to a bad citation, s/he must first cross-reference the citation in the answer with the 31 October 2019 legal basis.
      I understand that the EPO’s decision to permit 31 October 2018 legal basis in the EQE 2021 was intended to provide relief to EQE 2020 candidates, but I personally would have opted to use 31 October 2019 legal basis if I had known that I would be able to avoid these problems regarding the estimation and possible appeal of my results. I feel like the failure of the EPO to provide 31 October 2018 legal basis, alongside 31 October 2019 legal basis, in the Examiner’s report, combined with the failure of the EPO to warn candidates that the Examiner’s report would only contain one year’s legal basis, amounts to unfair treatment of candidates who were effectively encouraged to use the other year’s legal basis.
      Specifically, candidates who used 31 October 2019 legal basis are already better able to know how many points they need to win in appeal, and which points are likely to be won, since they can compare their citations with the citations in the Examiner’s report. In contrast, candidates who used 31 October 2018 legal basis have less certainty about how many points they need to win in appeal, and also which points they are likely to win, because they do not know as easily which citations are wrong. Thus candidates who used 31 October 2018 legal basis are already being treated unfairly with respect to candidates who used 31 October 2019 legal basis.
      I sincerely hope that the EPO will issue an updated Examiner’s report with both 31 October 2018 and 31 October 2019 legal basis as soon as possible, and in particular before any sort of appeal period begins to run. I also hope that the Boards of Appeal will be mindful of this difference of treatment when appeals are heard.

      Delete
    2. 31 October 2018? That is ancient history. You deserve to fail if you used that!

      "For the 2021 main examination only, candidates' answers will be marked either on the basis of the legal texts and document versions in force on 31 October 2019, or on the basis of those in force on 31 October 2020, depending on which of the two dates would give the candidate the higher mark"

      Delete
    3. I donot think that this D1 answer requires any legal basis that is different on 31 Oct 2019 compared to 31 October 2020: no relevant Rule changes, no relevant OJ publications in the intermediate year, no decisions that were published in the year.

      Maybe in A, B or C (e.g., the story with the ranges in C), but I did not check that.

      Delete
    4. With one exception: GL/PCT-EPO (Nov 2018), in force on 31.10.2019, did not yet have section A-V, which was only introduced in GL/PCT-EPO (Nov 2019), in force on 31.10.2020.

      GL/PCT-EPO (Nov 2018) does not discuss text translations of text matter in drawings at all.

      GL/PCT-EPO (Nov 2019 and Mar 2021) A-V, 8 "Text matter in drawings" includes:
      "Where any text matter of the drawings is filed in a language which is different from the language of the description and the claims, the receiving Office will invite the applicant to furnish a translation of the text matter of the drawings into the language in which the international application is to be published (Rule 26.3ter). The receiving Office decides whether the correction was submitted within the two-month time limit under Rule 26.2 and, if so, whether the international application so corrected is or is not to be considered withdrawn. However, no international application may be considered withdrawn for lack of compliance with the physical requirements referred to in Rule 11 if it complies with them to the extent necessary for the purpose of reasonably uniform international publication (Rule 26.5)."

      Delete
  32. Just saying...20 June 2021 at 22:17

    Please be informed:
    You messed up 2018, 2019 and 2020:
    For the 2021 main examination only, candidates' answers will be marked either on
    the basis of the legal texts and document versions in force on 31 October 2019, or
    on the basis of those in force on 31 October 2020, depending on which of the two
    dates would give the candidate the higher mark.

    ReplyDelete
  33. W.r.t. PCT Question 5, the Examiners' Report indicated "This question was generally well answered, but marks were often lost for not providing legal basis."

    However, compared to earlier years, the Committee seems to have been quite tolerant for legal basis. E.g., it was not required to cited R.12.1(a) PCT as legal basis in the PCT as to why Rule 157(2) EPC applies; nor was it required to give any legal basis for the EPO-WIPO Agreement, such as Art.16(4)(c) PCT (which was sometimes required in previous years, although most often not), or OJ 2017, A115 and updates.

    ReplyDelete
  34. W.r.t. the partial first application issue in Q.4-c), surprisingly the Examiners' Report only gives G 1/15, but does not cite T 282/12, which is exactly on this issue, and which is cited in the case Law Book (July 2019) II.D.4.1 and, in particular, II.D.5.3.3.

    See https://www.epo.org/law-practice/case-law-appeals/recent/t120282eu1.html:
    Catchwords: "For reasons of consistency, the rationale of decision G 1/15 (concept of partial priority) must also apply in the context of deciding whether an application from which priority is claimed is the first application within the meaning of

    Article 87(1) EPC. Indeed, just as a priority application and a patent claiming priority therefrom may partially relate to the same invention, the priority application and an earlier application filed by the same applicant may also partially relate to the same invention. In that case, the priority application would be the first application in respect of only that part of the invention which is not the same as in the earlier application (see points 2.1 to 2.7)."

    Case Law Book II.D.5.3.3:
    "In T 282/12 the board held that, for reasons of consistency, the rationale of decision G 1/15 (OJ 2017, A82) must also apply in the context of deciding whether an application from which priority is claimed is the first application within the meaning of Art. 87(1) EPC (see more detailed summary of this aspect of the decision in chapter II.D.4.1. above). The board also cautioned in this context against assessing priority by performing a test under Art. 123(2) EPC, which approach had led the opposition division to find that D1 was the first application. The assessment of priority and of the requirement of Art. 123(2) EPC were both based on the concept of disclosure. However, applying a test under Art. 123(2) EPC could lead to wrong conclusions in certain circumstances because the concept of "partial" validity of an amendment does not exist, whereas "partial priority" does. Relating this to the facts of the case (gap width of coated tablet), the board agreed that amending a range from 5% to 33% (i.e. the range in claim 1 and in the priority document D1) to 3% to 33% (i.e. the range in the proprietor’s earlier application D22) would result in the addition of subject-matter. However, both of these encompassed an identical part, i.e. the sub-range 5% to 33%, which defined alternative dosage forms that did not change their identity, whether they were claimed as such or as part of a broader group of compositions, including other compositions with a gap width outside the range 5% to 33%. Thus, in the present case, the mere application of an assessment pursuant to Art. 123(2) EPC to the assessment of the validity of the priority claim did not allow the conclusion that D1 and D22 related in part to the same invention and therefore that the priority was not valid over the whole scope of claim 1."

    ReplyDelete
  35. Update 21/6/2021: The results letters have been send out. The letters provide:

    "Due to the technical issues that occurred during the first flow of part one of paper D of the electronic EQE of 2021, the Examination Board has decided to exceptionally neutralize this part of the examination. Therefore, the Examination Board has exceptionally awarded to all candidates full marks (25 marks) for the flow D1-1, which consisted of questions 1, 2 and 6."

    The tables with marks attached to the results letters show the marks that were actually scored by the candidate (excluding the neutralization); the score shown in the letter itself includes the effect of the neutralization.

    ReplyDelete
  36. D 0026/21 () of 26.4.2022 (as far as relating to D1-2 of D 2021)

    Sachverhalt und Anträge

    I. Der Beschwerdeführer hat an der Hauptprüfung der Europäischen Eignungsprüfung ("EEP") 2021 teilgenommen. Mit Schreiben des Prüfungssekretariats vom 21. Juni 2021 hat der Vorsitzende der Prüfungskommission dem Beschwerdeführer mitgeteilt, dass letzterer u. a. für die Prüfungsaufgabe B die Punktzahl 32, für die Prüfungsaufgabe C die Punktzahl 25 und für die Prüfungsaufgabe D die Punktzahl 48 erreicht habe. Auf dieser Grundlage hat die Prüfungskommission entschieden, dass die Bedingungen des Artikels 14 (1) der Vorschriften über die europäische Eignungsprüfung für zugelassene Vertreter ("VEP"; Zusatzpublikation 2, ABl. EPA 2019, S. 2 ff.) nicht erfüllt seien und der Beschwerdeführer daher die EEP nicht bestanden habe.

    II. Mit Schreiben vom 2. August 2021, eingegangen beim Prüfungssekretariat per Fax am selben Tag und im Original am 4. August 2021, legte der Beschwerdeführer gegen die Entscheidung der Prüfungskommission vom 21. Juni 2021 Beschwerde ein. Er entrichtete zudem am 30. Juli 2021 die Beschwerdegebühr.

    III. Der Beschwerdeführer stellte folgende Anträge:
    (1) Unter Berücksichtigung besonderer Umstände sei die Bewertung der Bearbeitung der Teilaufgabe D1-2 der EEP des Beschwerdeführers erneut vorzunehmen und die Note BESTANDEN zu vergeben.
    (2) Hilfsweise zu Antrag (1) sei die Bewertung der Teilaufgabe D1-1 der Bearbeitung der EEP des Beschwerdeführers für die Gesamtwertung heranzuziehen und der Punkteausgleich von 25 Punkten für die Teilaufgabe D1-2 zu vergeben.
    […]

    IV. Die Beschwerdegründe lassen sich wie folgt zusammenfassen:
    a) Während der Bearbeitung des Aufgabenteils D1-2 am 2. März 2021 sei es mehrfach zu einem vom Beschwerdeführer unverschuldeten technischen Versagen der eingesetzten Prüfungssoftware "WISEflow" gekommen, so dass eine Fortsetzung der Bearbeitung währenddessen unmöglich gewesen sei. Eine Kontaktierung des Aufsichtspersonals, welches vergeblich versuchte, Hilfestellung zu leisten, habe zu keiner Abhilfe geführt, wie sich aus dem als Anlage 4 der Beschwerdebegründung beigefügten Chat-Protokoll vom 2. März 2021 ergebe. Im Ergebnis hätten dem Beschwerdeführer daher nur knapp 32 Minuten anstelle von 70 Minuten Bearbeitungszeit zur Verfügung gestanden. Im Rahmen des Chats habe jedoch nicht geklärt werden können, ob und wie es zu einer Kompensation käme. Eine Zeitverlängerung sei aus organisatorischen Gründen nicht gewährt worden. Außerdem sei die Bearbeitung der Aufgabe 4 in Folge eines Programmfehlers nicht gespeichert worden. Der Chat selbst erfülle aufgrund seiner Schriftform sowie der direkten Involvierung des Prüfungssekretariats die Anforderungen an eine Beschwerde über den Ablauf der Prüfung im Sinne von Regel 19 (3) ABVEP. Gleichwohl habe der Beschwerdeführer ausweislich der Anlage 3 noch am selben Tag der Prüfung per E-Mail eine Meldung im Sinne des Abschnitts Nr. 39 der "Anweisungen an die Bewerber für den Ablauf der europäischen Eignungsprüfung 2021" gemacht, hierauf jedoch in der Sache nie eine Stellungnahme erhalten. Es sei angemessen, angesichts der abnormalen Gesamtumstände unter Wahrung des Grundsatzes der Gleichbehandlung wie bei der Bewertung für D1-1 geschehen einen Punkteausgleich von 25 Punkten für die Teilaufgabe D1-2 zu vergeben. Auch für die Aufgaben 3 und 5 hätten unter Berücksichtigung der Umstände mehr Punkte vergeben werden müssen.
    […]

    V. Mit Schreiben vom 10. August 2021 teilte das Prüfungssekretariat dem Beschwerdeführer mit, dass die Prüfungskommission der Beschwerde nicht abgeholfen habe.

    VI. Mit Bescheid vom 18. Februar 2022 wurde dem Beschwerdeführer gemäß Artikel 14 der Ergänzenden Verfahrensordnung der Beschwerdekammer in Disziplinarangelegenheiten ("VOBKD"; Zusatzpublikation 1, ABl. EPA 2021, S. 67 ff.) die vorläufige Meinung der Kammer mitgeteilt und ihm Gelegenheit zur Stellungnahme binnen eines Monats gegeben. Der Beschwerdeführer hat hiervon keinen Gebrauch gemacht.

    VII. […]

    ReplyDelete
    Replies
    1. D 0026/21 [continued]

      Entscheidungsgründe

      1.-3. […]

      4. Antrag (1): Neuvornahme der Bewertung für Prüfungsaufgabe D1-2

      4.1 Der Antrag (1) des Beschwerdeführers ("Unter Berücksichtigung besonderer Umstände sei die Bewertung der Bearbeitung der Teilaufgabe D1-2 der EEP des Beschwerdeführers erneut vorzunehmen und die Note BESTANDEN zu vergeben") ist nicht eindeutig. Auf der einen Seite könnte das Begehren, die Bewertung der Teilaufgabe D1-2 "erneut vorzunehmen", so verstanden werden, dass entsprechend der ständigen Vorgehensweise der Kammern bei Vorliegen von schweren und eindeutigen Fehlern die Angelegenheit an die Prüfungskommission zur erneuten Entscheidung zurückverwiesen werden möge (s. o. Ziffer 3). Auf der anderen Seite wird im Antrag eine Zurückverweisung an die Prüfungskommission nicht explizit genannt und außerdem die Vergabe einer konkreten Note beantragt. Letzteres ist jedoch den Kammern grundsätzlich verwehrt und kann nur ganz ausnahmsweise geschehen, s. o. Ziffer 3. Da aber die beantragte erneute Vornahme der Bewertung eindeutig in Richtung der Prüfungskommission hinweist, welche ja bereits die erste Bewertung vorgenommen hat, legt die Kammer zugunsten des Beschwerdeführers den Antrag (1) so aus, dass nur die Zurückverweisung an die Prüfungskommission zur erneuten Entscheidung begehrt wird; der Antrag auf Vergabe der Note BESTANDEN ist dagegen lediglich als ein zeitlich nachgelagertes und nicht an die Kammer, sondern an die Prüfungskommission gerichtetes Begehren zu verstehen, über das daher die Kammer im Rahmen der Beschwerde nicht zu entscheiden hat.

      4.2 Die Kammer hat den Beschwerdeführer in ihrem Bescheid vom 18. Februar 2022 ausdrücklich auf ihre gerade dargelegte Auslegung seines Antrags hingewiesen. Der Beschwerdeführer hat hierzu keine Stellung genommen.

      4.3 Für eine Zurückverweisung müsste also ein schwerer und eindeutiger Fehler vorliegen, der eine Verletzung der Vorschriften der VEP oder der bei ihrer Durchführung anzuwendenden Bestimmungen oder höherrangiges Recht darstellt (s. o. Ziffer 2).

      Delete
    2. D 0026/21 [continued]

      4.4 In Betracht kommt hier eine Verletzung von Regel 19 (3) und (4) ABVEP.

      4.4.1 Regel 19 (3) ABVEP sieht vor, dass Beschwerden von Prüflingen über den Ablauf der Prüfung von der Prüfungskommission nur dann behandelt werden, wenn sie spätestens 30 Minuten nach dem Schlusssignal am letzten Prüfungstag gegenüber der leitenden Aufsichtsperson schriftlich unter Darlegung der Tatsachen vorgebracht worden sind. Gemäß Regel 19 (4) ABVEP müssen die gemäß Regel 19 ABVEP getroffenen Entscheidungen der Prüfungskommission alle Beweismittel berücksichtigen, mit Gründen versehen sein und schriftlich ergehen.

      4.4.2 Ergänzend sehen die "Anweisungen an die Bewerber für den Ablauf der europäischen Eignungsprüfung 2021" (ABl. EPA 2021, A13; im Folgenden: "Anweisungen") in Nummer 39 vor, dass Beeinträchtigungen während der Prüfung, die der Bewerber nicht zu verantworten bzw. zu vertreten hat, dem Prüfungssekretariat innerhalb von 24 Stunden nach dem offiziellen Ende der jeweiligen Prüfung (via E-Mail an helpdesk@eqe.org) mitgeteilt werden müssen.

      4.4.3 Der Beschwerdeführer hat vorliegend schwerwiegende Beeinträchtigungen des Prüfungsablaufs während der Prüfungsaufgabe D1-2 aufgrund Funktionsstörungen in der verwendeten Prüfungssoftware "WISEflow" geltend gemacht, die insbesondere zu einem deutlichen Zeitverlust, aber auch zum Verlust der bereits niedergelegten Antwort zu Aufgabe 4 geführt hätten. Diese Rüge ist ohne Weiteres als eine Beschwerde über den "Ablauf der Prüfung" im Sinne von Regel 19 (3) ABVEP einzuordnen. Für die Zulässigkeit der Rüge ist es dabei unerheblich, ob die Beeinträchtigungen tatsächlich vom Beschwerdeführer zu verantworten oder zu vertreten waren oder nicht, wie aber Nummer 39 der Anweisungen eventuell suggerieren könnte: Die vorrangige Vorschrift in Regel 19 (3) ABVEP sieht eine solche Einschränkung nämlich nicht vor. Die Frage, wer die geltend gemachten Beeinträchtigungen zu verantworten oder zu vertreten hat, wird vielmehr bei der von der Prüfungskommission zu treffenden inhaltlichen Entscheidung über die Begründetheit der Beschwerde eine Rolle spielen.

      4.4.4 Der Beschwerdeführer hat auch die formellen Voraussetzungen für das Einlegen einer Beschwerde im Sinne von Regel 19 (3) ABVEP erfüllt. Hierbei kann dahinstehen, ob - wie vom Beschwerdeführer geltend gemacht - bereits die Schilderung der Beeinträchtigungen im Rahmen eines elektronischen Chats mit dem Aufsichtspersonal während der Prüfung die Voraussetzung des "schriftlichen Darlegens gegenüber der leitenden Aufsichtsperson unter Darlegung der Tatsachen" im Sinne der genannten Vorschrift erfüllt. Jedenfalls hat er noch am selben Tag der Prüfung den in Nummer 39 der Anweisungen vorgeschriebenen Weg begangen und per E-Mail an das Prüfungssekretariat die Beeinträchtigungen dargelegt. Ob dabei eine Kommunikation per E-Mail das Schriftformerfordernis gemäß Regel 19 (3) ABVEP erfüllt, kann ebenfalls offenbleiben, da das Prüfungssekretariat den Kommunikationsweg über E-Mail in Nummer 39 der Anweisungen ausdrücklich eröffnet hat und sich somit auf einen Mangel der Schriftform nicht berufen könnte. Schließlich hat der Beschwerdeführer die Beschwerde noch am Tag der Prüfung und damit rechtzeitig im Sinne von Regel 19 (3) ABVEP ("spätestens 30 Minuten nach dem Schlusssignal am letzten Prüfungstag") eingereicht. Die insoweit in Nummer 39 der Anweisungen kürzer angesetzte Frist ("innerhalb von 24 Stunden nach dem offiziellen Ende der jeweiligen Prüfung") - die der Beschwerdeführer ohnehin ebenfalls eingehalten hat - kann im Übrigen die vorrangige Fristregelung gemäß ABVEP nicht abbedingen und wäre daher praktisch über den Vertrauensgrundsatz nur bei Beschwerden über den Verlauf der Prüfung am letzten Prüfungstag relevant.

      Delete
    3. D 0026/21 [continued]

      4.4.5 Die Prüfungskommission wäre daher gemäß Regel 19 (4) ABVEP verpflichtet gewesen, über die Beschwerde des Beschwerdeführers hinsichtlich der behaupteten Beeinträchtigungen während der Prüfung unter Berücksichtigung aller Beweismittel eine schriftliche und mit Gründen versehene Entscheidung zu treffen. Eine solche begründete Entscheidung ist jedoch zu keinem Zeitpunkt ergangen, weder durch gesondertes Schreiben der Prüfungskommission, noch im Rahmen des Schreibens vom 21. Juni 2021, mit dem dem Beschwerdeführer die von der Prüfungskommission vergebenen Noten mitgeteilt wurden, noch zusammen mit der Nichtabhilfeentscheidung mit Schreiben vom 10. August 2021.

      4.5 Die Beschwerde ist daher im Antrag 1 begründet und die angefochtene Entscheidung insofern aufzuheben. Im Rahmen ihrer Entscheidung gemäß Regel 19 (4) ABVEP wird die Prüfungskommission Gelegenheit haben, neben den vom Beschwerdeführer vorgelegten Beweismitteln gegebenenfalls auch weitere Beweismittel in ihrer Verfügungsgewalt (vgl. die vom Beschwerdeführer in seiner Beschwerde so genannten "Editionsbegehren" hinsichtlich der Nutzerprotokolle des Programms WISEflow mit der Nutzerkennung des Beschwerdeführers) gebührend zu berücksichtigen, um den Sachverhalt vollumfänglich zu würdigen. Sollte die Prüfungskommission dabei zu dem Ergebnis kommen, dass die Beschwerde im Sinne von Regel 19 (3) ABVEP unbegründet ist (und entsprechend auch die Bewertung der Prüfungsarbeit des Beschwerdeführers hinsichtlich der Prüfungsaufgabe D1-2 nicht zu ändern ist), muss sie dies entsprechend begründen; dem Beschwerdeführer stünde hiergegen erneut ein Beschwerderecht gemäß Artikel 24 (1) VEP offen. Sollte die Prüfungskommission dagegen zu dem Ergebnis kommen, dass die Beschwerde im Sinne von Regel 19 (3) ABVEP begründet ist, wird sie überdies Gelegenheit haben, einen nach Art und Ausmaß in der gegebenen Situation angemessenen Ausgleich für die Beeinträchtigungen zu finden und als eine mögliche Ausgleichsmaßnahme die Benotung hinsichtlich Prüfungsaufgabe D1-2 gemäß Artikel 6 (5) Satz 2 VEP entsprechend zu korrigieren. Der Beschwerdeführer kann dann gegebenenfalls im Rahmen einer erneuten Beschwerde die von der Prüfungskommission getroffenen Anordnungen überprüfen lassen.

      4.6 Soweit der Beschwerdeführer inhaltlich die Benotung der Teilaufgaben 3 und 5 der Prüfungsaufgabe D1-2 anzugreifen scheint, führt dies nicht ebenfalls zu einer Zurückverweisung der Angelegenheit an die Prüfungsabteilung. Insofern hat der Beschwerdeführer nämlich keinen schweren und eindeutigen Fehler geltend gemacht.


      5. Antrag (2): Hilfsweise Punkteausgleich von 25 Punkten für Prüfungsaufgabe D1-2

      Da der Antrag (1) in der Auslegung durch die Kammer (siehe oben Ziffer 4.1) bereits begründet ist, ist die Bedingung für die Behandlung des hilfsweise gestellten Antrags (2) ("Hilfsweise zu Antrag (1) sei die Bewertung der Teilaufgabe D1-1 der Bearbeitung der EEP des Beschwerdeführers für die Gesamtwertung heranzuziehen und der Punkteausgleich von 25 Punkten für die Teilaufgabe D1-2 zu vergeben") nicht erfüllt. Inhaltlich wäre letzterer entsprechend der Auslegung durch die Kammer ohnehin - wie das Begehren in Antrag (1), die Note BESTANDEN zu vergeben - inhaltlich an die Prüfungskommission nach Zurückverweisung der Angelegenheit gerichtet.

      6. -8. […]

      Delete
    4. D 0026/21 [continued]

      9. Ergebnis

      9.1 Die Beschwerde des Beschwerdeführers ist in seinen Anträgen (1) und (3), nicht aber im Antrag (5) begründet, so dass die Angelegenheit hinsichtlich der Prüfungsaufgaben D1-2 und B zur erneuten Entscheidung an die Prüfungskommission zurückzuverweisen ist.

      9.2 In der Konsequenz erscheint die Rückzahlung der kompletten Beschwerdegebühr als der Billigkeit entsprechend, Artikel 24 (4) Satz 3 VEP.

      10. – 11. […]

      Entscheidungsformel

      Aus diesen Gründen wird entschieden:

      1. Die angefochtene Entscheidung der Prüfungskommission wird aufgehoben.

      2. Die Angelegenheit wird hinsichtlich der Prüfungsaufgaben D1-2 und B zur erneuten Entscheidung an die Prüfungskommission zurückverwiesen.

      3. Die Rückzahlung der Beschwerdegebühr wird angeordnet.

      Delete

Post a Comment